Barb is three times as old as her twin nieces Allie and Sam. The sum of their three ages is 55. How old is each person now?

Answers

Answer 1

Answers: Each twin is 11 years old. Barb is 33 years old

Work Shown:

x = age of each twin

3x = Barb's age

x+x+3x = 5x = total of their ages = 55

5x = 55

x = 55/5

x = 11

Each twin is 11 years old. Barb is 3x = 3*11 = 33 years old.

Check: twin1+twin2+barb = 11+11+33 = 55

The answers are confirmed.


Related Questions

120 pens and pencils triple the number of pens than pencils how many pens

Answers

If number of pencils are triple the number of pens, then the number of pens is 30

Total number of pens and pencils = 120

Number of pencil is triple the number of pens

Consider the number of pens as x

Number of pencils = 3x

Sum of the pens and pencils = 120

Then the equation will be

Number of pens +  Number of pencils = Sum of number of pens and pencils

Substitute the values in the equation

x + 3x = 120

Add the like terms

(1 + 3)x = 120

4x = 120

Move 4 to the right hand side of the equation

x = 120 / 4

x = 30

Therefore, the number of pens is 30

Learn more about equation here

brainly.com/question/14603452

#SPJ4

rachel's coffee shop makes a blend that is a mixture of two types of coffee. type a coffee costs rachel per pound, and type b coffee costs per pound. this month, rachel made pounds of the blend, for a total cost of . how many pounds of type a coffee did she use?

Answers

Rachel's Coffee Shop used 33 pounds of type A coffee at a cost of $4.20 per pound, and four times as many pounds of type B coffee at a cost of $5.50 per pound, to create a blend costing a total of $864.60.

Let's use the variable "x" to represent the number of pounds of type A coffee used in the blend.

The amount of type B coffee used in the mix was four times that of type A, therefore we can write:

number of pounds of type B coffee = 4x

The total cost of the blend is given as $864.60, so we can write an equation based on the cost:

[tex]4.2x + 5.5(4x) = 864.60[/tex]

Simplifying and solving for x, we get:

[tex]4.2x + 22x = 864.60[/tex]

[tex]26.2x = 864.60[/tex]

[tex]x = 33[/tex]

As a result, the mix contained 33 pounds of type A coffee.

To check our answer, we can substitute x = 33 into the equation we used to solve for it and verify that the total cost is indeed $864.60.

[tex]4.2(33) + 5.5(4*33)[/tex]

= [tex]138.6 + 726.0[/tex]

= [tex]864.6[/tex]

The total cost matches, so our answer is correct.

Therefore, Rachel's Coffee Shop used 33 pounds of type A coffee in the blend.

Learn more about equation here:

https://brainly.com/question/29174899

#SPJ4

The complete question is:

A blend made by Rachel's Coffee Shop combines two different kinds of coffee. Rachel pays $4.20 per pound for type A coffee and $5.50 for type B coffee. This month's blend cost $864.60 and included four pounds more of type B coffee than type A. How many pounds of coffee of type A were consumed?

Ticket lines are huge around the Colossal Arena. The Boulders, one of the most popular rock groups ever, are giving a concert on Saturday. When the tickets went on sale Monday, 500 were sold in the first hour, 560 tickets were sold in the second hour, and 640 tickets were sold in the third hour. After the second hour, the increase in ticket sales every hour was 20 more than the increase of the previous hour. If sales continue at that rate, in what hour will 10,000 tickets be sold for the Boulders concert?Explain your answer in detail

Answers

If after the second hour, the increase in ticket sales every hour was 20 more than the increase of the previous hour, it will take 12 hours to sell 10,000 tickets for the Boulders concert.

To solve this problem, we need to find the number of hours it will take to sell 10,000 tickets for the concert. We are given the number of tickets sold in the first three hours and the rate at which ticket sales increase each hour after the second hour.

Let's denote the number of tickets sold in the fourth hour by "x". Then, the number of tickets sold in the fifth hour will be "x + 20", and the number of tickets sold in the sixth hour will be "x + 40", and so on.

Using this information, we can set up an equation to find "x":

500 + 560 + x + (x + 20) + (x + 40) + ... = 10,000

Simplifying the equation, we get:

3x + 1020 + 20n = 9500

where "n" is the number of hours after the third hour.

Solving for "n", we get:

n = (10,000 - 1020 - 3x) / 20

Now, we need to find the value of "x" that satisfies this equation. We can do this by testing different values of "x" until we find the one that gives an integer value for "n".

Let's start by trying "x = 640 + 20 = 660". Plugging this value into the equation, we get:

n = (10,000 - 1020 - 3(660)) / 20 = 9

We arrived at this solution by using the given information about the rate of increase in ticket sales and setting up an equation to solve for the number of hours it will take to sell 10,000 tickets. We then tested different values of "x" until we found the one that satisfied the equation and gave an integer value for "n".

To learn more about time click on,

https://brainly.com/question/16633159

#SPJ4

Use your understanding of transformations and key features to write the slope-intercept equation of this linear function.
Enter the correct answer in the box by replacing the values of m and b.

Answers

The slope-intercept equation of this linear function is y = -3x + 5.

What is the point-slope form?

Mathematically, the point-slope form of a straight line can be calculated by using this mathematical expression:

y - y₁ = m(x - x₁) or y - y₁ = (y₂ - y₁)/(x₂ - x₁)(x - x₁)

Where:

m represents the slope.x and y are the points.

Based on the information provided, we can logically deduce the following data points on the line:

Points on x-axis = (0, 1).Points on y-axis = (5, 2).

At point (0, 5), a linear equation of this line can be calculated in slope-intercept form as follows:

y - 5 = (2 - 5)/(1 - 0)(x - 0)

y - 5 = -3(x - 0)

y = -3x + 5

Read more on slope here: brainly.com/question/3493733

#SPJ1

A stereo that regularly sells for $330 is on sale at 15% off. How much will a customer save on the stereo during the sale?

Answers

Answer: 49.5 dollars

Step-by-step explanation: the original price is 330, but the sale takes 15 percent off

so we turn 15% into a decimal so we can multiply 330 and 15%

we get 0.15 × 330 = 49.5

Answer please! 50 Points!

Answers

Option A is correct

The proportion of twins born in a town is p = 0.12. Suppose we randomly select 100 women from this town who give birth in the next year. Which of the following is the mean of the sampling distribution of p hat ?

Mu Subscript p hat = p = 0.12
Mu Subscript p hat = n p = 100 (0.12) = 12
Mu Subscript p hat = 1 minus p = 1 minus 0.12 = 0.88
Mu Subscript p hat = n (1 minus p) = 100 (1 minus 0.12) = 88

Answers

The sampling distribution of p hat follows a normal distribution thus, the value of Mu Subscript p hat = p = 0.12. Option 1 is the correct answer.

What is normal distribution?

A probability distribution that is symmetric about the mean is the normal distribution, sometimes referred to as the Gaussian distribution. It demonstrates that data that are close to the mean occur more frequently than data that are far from the mean.

The most typical form of distribution assumed in technical stock market analysis and other statistical investigations is the normal distribution. The mean and the standard deviation are the two variables that make up the standard normal distribution.

The sampling distribution of p hat follows a normal distribution thus,

Mu Subscript p hat = p

Here, the value of p = 0.12

Hence, the value of Mu Subscript p hat = p = 0.12. Option 1 is the correct answer.

Learn more about normal distribution here:

https://brainly.com/question/29509087

#SPJ1

Answer:

A. Mu Subscript p hat = p = 0.12

Step-by-step explanation:

Louise is planning to renovate her house. She intends to spend no more than $30 000. She has
$20 000 to invest in an account that pay 4.28% compounded monthly. How long will it take
Louise to meet her goal? Show your work. Round your answer to the nearest tenth of a year.

Answers

Answer: To solve this problem, we can use the formula for calculating the future value of an investment:

FV = PV * (1 + r/n)^(nt)

where:

FV is the future value of the investment

PV is the present value of the investment (the initial amount Louise has to invest)

r is the annual interest rate (4.28%)

n is the number of times the interest is compounded in a year

t is the number of years the investment is made

Since the interest is compounded monthly, we need to convert the annual interest rate to a monthly rate:

r/12 = 4.28% / 12 = 0.3566666666666667%

Now, we can use the formula to calculate the future value of Louise's investment:

FV = $20,000 * (1 + 0.3566666666666667%)^(12t)

We can use trial and error or iterative methods to find the value of t that satisfies the condition: FV = $30,000.

A quick approximation is to use the formula for simple interest:

FV = PV * (1 + r * t) = $20,000 * (1 + 0.0428 * t)

t = ($30,000 - $20,000) / ($20,000 * 0.0428) = 2.857142857142857 years, or approximately 2.9 years.

This is an approximation, but it should be close to the actual answer. To get a more accurate answer, we can use an iterative method, such as Newton-Raphson or bisection, to solve for t in the formula above.

Step-by-step explanation:

The number b is irrational. Which statement about
41+b is true?

Answers

An irrational number is created by adding two rational numbers, according to the answer to the given question. If the oppositely irrational numbers a and b.

What is rational numbers?

Rational numbers are those that may be written as a ratio (or fraction) of two integers. A fraction with a non-zero denominator is said to be rational.

A few instances of rational numbers are 1/2, 1/5, and 3/4. A rational number, “0,” can also be stated in a number of different ways, such as 0/1, 0/2, and 0/3. But 1/0, 2/0, 3/0, and so forth.

The number seven makes sense. A rational number is produced when two integers are split. To get the rational number 7, divide the whole number 7 by the whole number 1. Since 7 can be obtained by dividing two integers, it is referred to as a rational number.

Two rational integers added together yield an irrational number. Ab is irrational if and only if a and b are mutually exclusive irrational numbers.

Therefore, a = 2 and b = ab = 2 is an irrational number.

Learn more about rational numbers here:

https://brainly.com/question/24398433

#SPJ4

At hannah’s school 3/5 of the school playground is covered with grass and 5/8 of the grassy area is a baseball field. How much of the school playground is taken up by the baseball field

Answers

The baseball field takes up 3/8 or 37.5% of Hannah’s school playground. This means that the remaining 62.5% of the playground is taken up by other materials and the grassy area not occupied by the baseball field.

Hannah’s school playground is divided into two parts, with the majority of it being covered in grass. Specifically, 3/5 of the playground is grassy and the remaining 2/5 is taken up by other materials such as pavement or gravel. Of the grassy area, 5/8 is taken up by the baseball field. This means that the baseball field occupies a total of 15/40 or 3/8 of the entire playground.

In terms of proportions, if we consider the entire playground to be 100%, then the amount of the playground taken up by the baseball field is 3/8, or 37.5%. This means that 62.5% of the playground is taken up by the other materials and the remaining grassy area not occupied by the baseball field.In terms of area, the total size of the playground can be determined by multiplying the length of the playground by its width. If we assume that the length and width are equal, then the area of the playground can be calculated as the length squared. The area of the baseball field can then be calculated by multiplying 3/8 by the total area of the playground. This will give us the total area of the baseball field.

Overall, the baseball field takes up 3/8 or 37.5% of Hannah’s school playground. This means that the remaining 62.5% of the playground is taken up by other materials and the grassy area not occupied by the baseball field.

Learn more about playground here:

https://brainly.com/question/20036219

#SPJ4

Alexa is saving up to buy a new jacket. She already has $45 and can save an
additional $7 per week using money from her after school job. How much total
money would Alexa have after 8 weeks of saving? Also, write an expression that
represents the amount of money Alexa would have saved in w weeks.

Answers

Answer:

Step-by-step explanation:

Answer: 101

because if you multipled 7 times 8 you will get 56 and if you add it with 45 you will get 101

i hope this help

Final answer:

After 8 weeks of saving, Alexa will have a total of $101. The expression to show the money Alexa saves in 'w' weeks is 45 + 7w.

Explanation:

The subject of this question is

Mathematics

and it is geared towards

Middle School

grades. To solve this problem, we need to use

algebraic expressions

. Based on the problem given, Alexa has $45 initially and saves $7 per week additionally from her after school job. So if she saves for 8 weeks, Alexa would have accumulated an additional $7 times 8 = $56. Therefore, the total amount of money Alexa would have after 8 weeks is her initial amount of $45 plus the additional $56, which equals $101. To represent this in terms of an expression, where 'w' stands for weeks, you can write 45 + 7w. This expression lets us calculate the total savings for any number of weeks 'w'.

Learn more about Algebraic Expressions here:

https://brainly.com/question/34192827

#SPJ2

how many solutions do 2(x-4)=2x-8 have

Answers

The equation 2(x - 4) = 2x - 8 has infinitely many solutions.

What is an Equation?

An equation is the statement of two expressions located on two sides connected with an equal to sign. The two sides of an equation is usually called as left hand side and right hand side.

Given equation is 2(x - 4) = 2x - 8.

Solving this,

Using the distributive property, a (b + c) = ab + ac, 2(x - 4) = (2 × x) - (2 × 4).

Substituting in the original equation,

(2 × x) - (2 × 4) = 2x - 8

2x - 8 = 2x - 8

2x - 2x = -8 + 8

0 = 0

Right hand side and left hand side are equal if we put any value for x.

So it has infinitely many solutions for x.

Hence the given equation has infinitely many solutions.

Learn more about Equations here :

https://brainly.com/question/12222592

#SPJ9

8x^{2}-8x8x
2
−8x from -6x^{2}-10x+4−6x
2
−10x+4.

Answers

The value of the equation 8x² - 8x + 2 -( 6x² - 10x + 4) is 2x² + 2x - 2.

How to illustrate the equation

It should be noted that an equation simply has to do with the statement that illustrates the variables given. In this case, it is vital to note that

two or more components are considered in order to be able to describe the scenario.

It is important to note that an equation is the mathematical statement which can be made up of two expressions which are connected by an equal sign.

The equation can be illustrated thus:

8x² - 8x + 2 -( 6x² - 10x + 4)

8x² - 8x + 2 - 6x² + 10x - 4

= 2x² + 2x - 2

Learn more about equations on:

brainly.com/question/2972832

#SPJ1

Lena estimate is there a bottle contains 500 mm of water but actually contains 473 mm of water what is the percent error and Lena's estimate rounded to the nearest 10th of a percent HURRY FAST PLS!!!

Answers

5.4%   is the percent error .

What is the formula for errors?

The difference between the estimated value and the actual value in relation to the actual value is given as a percentage and is known as the percent error.

                             The relative error is multiplied by 100 to calculate the % error, in other words. The absolute value of the difference between the measured value and the actual value is divided by the actual value, then multiplied by 100 to get the percent error. 

 = 500 - 473/500  * 100

 =   27/500 * 100

 =   5.4%

Learn more about the percent error

brainly.com/question/29107960

#SPJ1

A hotel swimming pool measures 20 m by 8m.The hotel manager wants to put a 2 m wide deck around the pool. Caluculate the area of the deck.

Answers

128m

you can separate each section of the deck using a picture and find the areas of each that way. for example; there are 4 corner pieces with an area of 2x2 (4) each, 2 long sections with an area of 2x20 (40), and two shorter sections with an area of 8x2 (16)

What is the area of ∆AOB?

Answers

The Area of Rhombus is 97.5 cm².

What is area of Rhombus?

The area of a rhombus is equal to half the product of the lengths of the diagonals. The formula to calculate the area of a rhombus using diagonals is given as, Area = (a × b )/2 where, a and b are the diagonals of the rhombus.

Given:

As, diagonals of rhombus bisect perpendicularly.

so, In triangle OAD using Pythagoras theorem

AD² = OD² + OA²

10² = 7.8² + OD²

OD= 6.25 cm

So, length of BD = 2 x 6.25 = 12.5 cm

Now, area of Rhombus = 1/2 x a x b

                                       = 1/2 x 15.6 x 12.5

                                       = 97.5 cm².

Learn more about Area of Rhombus here:

https://brainly.com/question/30577072

#SPJ9

Use the following data sets to answer the question.
Set {12, 9, 30, 1, 15, 10,
A:
12}
Set
B:
9, 30, 12, 3, 16, 11,
18}
Which statement about the shape of the data is true?
Both sets have the same median, but Set A has a larger mean.
O Both sets have the same median, but Set B has a larger mean.
O Set B has a larger mean and median than Set A.
O Set A has a lower median but a higher mean than Set B.

Answers

Answer:

Both have the same median but set A has a larger mean.

Step-by-step explanation:

You're welcome.

Answer:
A
Step-by-step explanation:
Each set has a mean of 12, But the mean of set A is 12.71 and the mean of set b is 14.14 so the answer would be A.

At MeDonald's four cheeseburgers and three medium fries have a total of 2290 calories.Six cheeseburgers and two medium fries have 2560 calories. How many calories does each
Item contain?

Answers

Answer:

350 calories

Step-by-step explanation:

see the attachment.

Calories in one cheeseburgers = 135

And, Calories in one medium fries = 583

We have to given that,

At McDonald's four cheeseburgers and three medium fries have a total of 2290 calories.

And, Six cheeseburgers and two medium fries have 2560 calories.

Let us assume that,

Calories in one cheeseburgers = x

And, Calories in one medium fries = y

Hence, We get;

4x + 3y = 2290  . (i)

6x + 3y = 2560 .. (ii)

From (i);

3y = 2290 - 4x

Substitute above value in (ii);

6x + (2290 - 4x) = 2560

6x - 4x + 2290 = 2560

2x = 2560 - 2290

2x = 270

x = 270 / 2

x = 135

From (ii);

6 x 135 + 3y = 2560

810 + 3y = 2560

3y = 2560 - 810

3y = 1750

y = 1750/3

y = 583

Therefore, Calories in one cheeseburgers = 135

And, Calories in one medium fries = 583

Learn more about systems of equations at:

brainly.com/question/14323743

#SPJ6


a= 340 rounded to the nearest 10
b= 49.8 rounded to 1 DP
Find the minimum of a × b

Answers

Answer: First, we'll round the values of A and b:

A = 340 rounded to the nearest 10 = 340

b = 49.8 rounded to 1 decimal place = 49.8

Next, we'll find the product of A and b:

A × b = 340 × 49.8 = 16872

Finally, we'll find the minimum of A × b:

Minimum of A × b = 16872

So, the minimum value of A × b is 16872.

Step-by-step explanation:

11) Dalton calls a local moving company to determine how much it will cost him to move. If he hires 2 movers and rents a truck for an hour it costs $65. It costs $80 to hire 3 movers and rent a truck for an hour. What is the hourly rate to hire a mover? What is the hourly rate to rent the truck?

Answers

Answer: $35.

Step-by-step explanation: Let A = 2 movers and one truck for hour = $65

B = 3 movers and one truck for hour = $80

So, 1 mover = B - A

                  = $80 - $65

                  = $15

Hence, 2 movers cost = 2 × $15

                                  = $30

SO, rent of the truck hourly = A - cost of 2 movers

                                          = $65 - $30

                                          = $35

Therefore cost of renting the truck for an hour = $35

hi can i have help with this question please i can’t figure out what c is

Answers

Answer: Decagon

Step-by-step explanation:

That would be a decagon because it has 10 sides. It does not matter if the shape looks wonky.

Hope this helps :)

what is the problem from y=x+1

Answers

Answer: Change x to a number, and that number + 1 will be y

Step-by-step explanation: the reason for this is simple, change the independent variable, which is always in the problem, and you can figure out the dependent variable.

Answer:

X +1

Step-by-step explanation:

the graph is x +1.....

1589

Calculator

120°

126

What is the value of x?

125

139

Enter your answer in the box

121°

Answers

Answer:aaaa

Step-by-step explanation:because

Need answers stat trigonometry

Answers

The required measure of x is given in each triangle are given as,
2. x = 74.01     3.   25.09      4.  x = 23.78
5. x = 26.64    6. x = 21.03°  7. x = 39.55°

What are trig ratios?

If you know the lengths of two sides of a right triangle, you can use trigonometric ratios to calculate the measures of one (or both) of the acute angles.

Here,
2.
For the given triangle
Apply the trigonometric ratios,

Tan 62 = x/25
x = 25 tan62
x = 74.01

Similarly,
3. x = 11/sin26  = 25.09
4. x = 32sin48 = 23.78
5. x = 29/cos12 = 29.64
6. cosx = 14/15 = 21.03°
7. tanx = 19/23 = 39.55°

Learn more about trig ratios here:

https://brainly.com/question/14977354

#SPJ1

the product 2√3 and 3√12 in simplified form is

Answers

The product of 2√3 and 3√12 in simplified form is 6√36. To simplify this expression, we can first multiply 2 and 3 to get 6. Then, we can simplify the square root of 3 and the square root of 12 by finding their prime factorizations:

√3 = √(3 * 1) = √3

√12 = √(12 * 1) = √(2^2 * 3) = 2√3

So, 2√3 = 2 * √3 = 2 * √3

And, 3√12 = 3 * 2√3 = 6√3

Finally, the product of 2√3 and 3√12 is 6 * 6√3 = 6√36.

Help please Hurry

What conclusion can be drawn about the number of trials and the probability of the coin landing on heads or tails? (1 point)
O a
О b
• c
O d
The experimental probability is closer to the theoretical probability for group Y than group X.
The experimental probability and the theoretical probability for group X is the same.
The experimental probability and the theoretical probability for group Y is the same.
The experimental probability is closer to the theoretical probability for group X than group Y.

Answers

The conclusion that can be drawn about the number of trials and the probability is A. The experimental probability is closer to the theoretical probability for group Y than X.

How to find the probability ?

The theoretical probability for picking a heads or tails is 50 % as there are only two outcomes of heads and tails.

The experimental probability for heads is 53 % in Group Y and 47 % for tails. This is close enough to the 50 % theoretical probability.

For group X, the experimental probability for heads for Group x is :

= 33 / 50

= 66 %

This is too far from the theoretical probability.

Find out more on theoretical probability at https://brainly.com/question/1503266

#SPJ1

The altitude of an airplane coming in for a landing is represented by the equation shown below, where y represents the altitude, in feet, of the airplane
and x represents the number of minutes the plane has been descending:
y=-5x + 100
Part A:
Create a table for the values when x = 0, 5, 8, 10, 30.
Include worked-out equations used to identify the values within the table.
Part B:
Identify the altitude after 5 minutes and after 30 minutes. Use 1-2 sentences to explain the altitude at these two times and describe what is happening
to the airplane at these time intervals.

Answers

Answer:

32x95%?

Step-by-step explanation:

Im so confused with this question statement.

what's the geometric mean between 3√5 and 6√5

Answers

Answer:

3√10

Step-by-step explanation:

[tex]GM=\sqrt{(3\sqrt{5})(6\sqrt{5})}=\sqrt{18*5}=\sqrt{90}=3\sqrt{10}[/tex]

Answer:   [tex]3\sqrt{10}[/tex]

Work Shown:

Part 1

[tex]g = \text{geometric mean of x and y}\\\\g = \sqrt{\text{x}\text{y}}\\\\g = \sqrt{3\sqrt{5}*6\sqrt{5}}\\\\g = \sqrt{(3*6)*(\sqrt{5}*\sqrt{5})}\\\\g = \sqrt{18\sqrt{5*5}}\\\\g = \sqrt{18\sqrt{5^2}}\\\\g = \sqrt{18*5}\\\\g = \sqrt{90}\\\\[/tex]

Part 2

[tex]g = \sqrt{90}\\\\g = \sqrt{9*10}\\\\g = \sqrt{9}*\sqrt{10}\\\\g = 3\sqrt{10}\\\\[/tex]

Now look at the exponential expression you found to be equivalent to 100,000 • 1/100,000 can you use addition or multiplication on the powers of this exponential expression to simplify it if not which operation works and which one doesn’t

Answers

The expression simplifies to 1, which means that 100,000 • 1/100,000 is equal to 1.

The exponential expression that is equivalent to 100,000 • 1/100,000 is:

[tex](10^5) * (10^-5)[/tex]

To simplify this expression using addition or multiplication on the powers, we can use the property of exponents that states:

[tex]a^m *a^n = a^{(m+n)[/tex]

This means that when we multiply two numbers with the same base, we can add their exponents to simplify the expression. However, this property does not work for division, as we need to subtract the exponents instead.

Using this property, we can simplify the expression as follows:

[tex](10^5) * (10^-5) = 10^{(5-5)} = 10^0 = 1[/tex]

Therefore, the expression simplifies to 1, which means that 100,000 • 1/100,000 is equal to 1. We can see that in this case, addition and multiplication do not help us simplify the expression, but we can use the exponent property to do so.

Learn more about exponential :

https://brainly.com/question/26540624

#SPJ4

1. An awning that covers a sliding glass door that is 88 inches tall forms an angle of 50' with the wall.
The purpose of the awning is to prevent sunlight from entering the house when the angle of elevation
of the Sun is more than 65'. See the figure. Find the length of L of the awning. Round to the nearest
Inch.

Answers

To calculate the length of the awning, we can use the law of cosines. The law of cosines states that in a triangle, the sum of the squares of the lengths of two sides is equal to the square of the length of the third side. In this case, we can let the angle of elevation of the sun be A, the length of the awning be L, and the angle formed by the awning and the wall be B. We then have:

L^2 = 88^2 + (65 * tan(A))^2 - 2(88)(65 * tan(A))cos(B)

Substituting in the given values for A, B, and L, we get:

L^2 = 88^2 + (65 * tan(65))^2 - 2(88)(65 * tan(65))cos(50)

Solving for L, we get a value of L = 103.9 inches. Rounding to the nearest inch, the length of the awning is 104 inches.
Other Questions
Find the values of x, y, and z When the insured purchased his health policy he was a window washer. He has since changed occupations and now manages a library. If the insurer is notified of the insured's change of occupation, the insurer shouldAdjust the benefit in accordance with the decreased riskReplace the policy with a new oneReturn any unearned premiumIncrease the premium Which graph represents the scenario of a woman who climbs a hill at a steady pace and then starts to run down the side?Graph IIGraph IIIGraph IGraph IV A roll of gasket material is 9 in wide. What length is needed to obtain 19 sq ft of the material? (Careful: The numbers are not expressed in compatible units.)fA length of ft is needed to obtain 19 sq ft of gasket material.(Type an integer, proper fraction, or mixed number.) help me pls thanks body paragraph about the vanishing island by Anya Groner cushman company had $820,000 in sales, sales discounts of $12,300, sales returns and allowances of $18,450, cost of goods sold of $389,500, and $282,080 in operating expenses. gross profit equals: 6-90. Evaluate the expression 10-2x for the x-values given below.(show work)a. x=2b. x = 1/2c. x = -2 A college cafeteria is looking for a new dessert to offer its 4,000 students. The table shows the preference of 225 students. Ice Cream Candy Cake Pie Cookies 81 9 72 36 27 Which statement is the best prediction about the slices of pie the college will need? The college will have about 480 students who prefer pie. The college will have about 640 students who prefer pie. The college will have about 1,280 students who prefer pie. The college will have about 1,440 students who prefer pie. What interesting feature did Albea. Each floor uses the same cob. Each floor has a different typSome of the floors do no conC. 1. a charge of -2.50 nc and a charge of -4.00 nc are placed 50.00 mm apart. find the resultant force on a charge of 8.00 nc placed 20.00 mm from the -2.50 nc charge and 30.00 mm from the -2.50 nc charge. RECAP NEED HELP PLEASE HELP!!!!!!!!!!!!!!!!!!!!!!!!!! What percentage of earth is covered in snow and ice? how did land and climate features influence bedouin family life? What is the significance of the following line?"He [the old man] was too simple to wonder when he had attained humility. But he knew he had attained it and he knew it was not disgraceful and it carriedno loss of true pride."O Highlights characteristics of Hemingway's Code HeroO Supports the idea that Santiago has lost touch with realityO Emphasizes Santiago's poornessO Foreshadows Santiago's ultimate downfall Please help me................................. This net will be folded to form a prism.A net of a rectangular prism. From top to bottom, the rectangles are labeled B, D, E, F. The left side is labeled A and the right side is labeled C.Which pairs of faces will be opposite one another when the prism is created? Check all that apply.D and FA and CA and BB and FB and EB and CE and FA and F ______ is a person, who could be an employee of a company, or a government agency, disclosing information to the public or some higher authority about any wrongdoing, which could be in the form of fraud, corruption, etc. Which is the correct order of the steps in a generalized signal transduction pathway?Signal molecule binds to receptorMolecules within cell transduce signalCell responds A woman normally weighs 125 lb. If she is standing on a spring scale in an elevator that is traveling downward, but slowing down, the scale will read.